Matematik

Vink til et bevis?

17. marts 2019 af Sejensscience - Niveau: Universitet/Videregående

Jeg ved virkelig ikke hvordan jeg skal starte med dette bevis, er der nogen der har et vink der kan hjælpe mig igang?


Brugbart svar (0)

Svar #1
18. marts 2019 af oppenede

Det gælder pr. definition hvis der for ethvert ε > 0 eksistere N, så x > N ⇒ |L - g(x)| < ε.

Lad ε > 0 være givet. Da L er supremum for g, findes der et x0 så |L - g(x0)| < ε.
|L - g(x)| = L - g(x) ≤ L - g(x0) = |L - g(x0)| < ε     Q.E.D.
             (1)           (2)             (1)
 1. Da L er supremum for g.
 2. ∀x > x0 da -g er svagt aftagende.


Brugbart svar (0)

Svar #2
18. marts 2019 af AskTheAfghan

Benyt, at

\begin{align*} L=\sup_{x\in [0,\infty)}g(x)\quad\stackrel{def} \iff\quad & \forall x\in [0,\infty):g(x)\leq L \quad \wedge \\ & \forall \epsilon>0\exists y\in [0,\infty):L-\epsilon<g(y) \end{align*}


Brugbart svar (0)

Svar #3
18. marts 2019 af Matematikerens

Så hvis jeg følger bevisskitsen i #1 kan jeg bruge  din defintion af supremum til at konkludere at g(x)-->L for

x-->uendelig fordi hvis man isolerer ε  i den nederste sætning i din defintion får man L - g(y) < ε hvilket svarer til resultatet i bevisskitsen i #1?


Brugbart svar (0)

Svar #4
20. marts 2019 af AskTheAfghan

#3

Da L = supx∈[0, ∞) g(x), vil g(x) ≤ L for ethvert x∈[0, ∞), og for ethvert ε > 0, vil der findes et y∈[0, ∞) sådan at L - ε < g(y). Da g er voksende, er L - ε < g(y) ≤ g(x) ≤ L for alle x ≥ y. Dette medfører, at |g(x) - L| < ε for alle x ≥ y.


Skriv et svar til: Vink til et bevis?

Du skal være logget ind, for at skrive et svar til dette spørgsmål. Klik her for at logge ind.
Har du ikke en bruger på Studieportalen.dk? Klik her for at oprette en bruger.